LSAT and Law School Admissions Forum

Get expert LSAT preparation and law school admissions advice from PowerScore Test Preparation.

 dhaozh
  • Posts: 10
  • Joined: Nov 05, 2011
|
#2822
Hello! The question stimulus concerns three studies having to do with night-lights and nearsightedness in children. I'm struggling to comprehend why the correct answer is D, not A. While you explain this, please also comment on the exact meaning of the author's conclusion author's conclusion, as I suspect that it might be the source of difficulty for me. Also, is it right to chalk up A's incorrectness simply to its method of reasoning, i.e. citing a new study to discredit the studies discussed in the stimulus? In other words, can this method of reasoning ever successfully weaken an argument based on experimental studies?

Thanks a ton for your help!
User avatar
 Dave Killoran
PowerScore Staff
  • PowerScore Staff
  • Posts: 5853
  • Joined: Mar 25, 2011
|
#2827
Hi David,

The conclusion in this argument is an interesting one, containing both conditional and causal elements. Conditionally, the conclusion posits that if a causal relationship does exist (and the argument references studies that counter that claim), then that causal relationship is cancelled out as the children age.

Answer choice (A) uses, as you note, a fourth study. Can this approach ever be effective in a Weaken question? Certainly. If an answer contained new evidence based on a proper study, and those results contradicted something in the stimulus, then it would weaken the argument. However, they don't often use this approach as the right answer because for it to work, they have to be pretty clear that the new study contradicts some portion of the stimulus. That tends to be fairly obvious, and thus relatively easy for students to spot.

So, turning to the content of this answer, what's the problem with it? Simply put, it doesn't address the "effects disappear with age" portion. The author already is suspicious of the causal relationship, so an answer that says that the causal relationship doesn't exist will not weaken the author's position--he or she would simply reply, "yeah, that doesn't surprise me." From an analogy standpoint, (A) is like hearing a statement such as, "If by some incredible chance I win the lottery, then I will give away all of the money to charity," and then replying, "You aren't going to win the lottery."

Answer choice (D) calls into question the "effects disappearing" portion by undermining the studies that the author used to make the connection that the nearsightedness disappears as children get older.

Please let me know if that helps. Thanks!
 dhaozh
  • Posts: 10
  • Joined: Nov 05, 2011
|
#2867
This is extremely helpful, Dave. Much appreciated!

Dave
 brittany1990
  • Posts: 21
  • Joined: Jul 11, 2012
|
#6665
Hi!

After reading the stimulus and question stem, I identified it as a weaken question. I eliminated A-C and was left with D and E. I picked E, but I do not understand why D is correct.

Thanks,
Brittany
 Steve Stein
PowerScore Staff
  • PowerScore Staff
  • Posts: 1153
  • Joined: Apr 11, 2011
|
#6667
Hey Brittany,

In that one, the doctor discusses one study in which children with nightlights were more likely to be nearsighted, and two subsequent studies, the subjects of which were older children, showed no such correlation.

The doctor's conclusion: the studies suggest that if nightlights cause nearsightedness, that effect disappears with age.

Answer choice D weakens the argument by removing the two subsequent studies as credible evidence. If both of the later studies had insufficient samples, then that leaves less reason to believe that any nearsighted effects actually disappear over time.

I hope that's helpful--let me know--thanks!

~Steve
 ioannisk
  • Posts: 6
  • Joined: Aug 20, 2013
|
#12349
Could someone explain how "A" is wrong?

I feel it is a candidate as it shows that nightlights don't affect at any age, and if it doesn't affect at any age, it doesn't "disappear" with age.
 Jacques Lamothe
PowerScore Staff
  • PowerScore Staff
  • Posts: 50
  • Joined: Sep 24, 2013
|
#12357
Hey ioannisk,

The problem with (A) is that the mentioned study does not show that night-lights do not affect children of any age. The study shows that the vision of infants currently sleeping with night-lights is not affected. That does not mean that by the time the infants become children who no longer use night-lights, their vision will remain unaffected. That's why answer choice (A) does not weaken the doctor's conclusion that night-light use during infancy causes near-sightedness in children that disappears with age.

I hope that helps! Let me know if anything I said is unclear.

Jacques
 reop6780
  • Posts: 265
  • Joined: Jul 27, 2013
|
#14739
The correct answer is D while I struggled between A and E.

The conclusion is a bit complex as it contains both conditional and causal statement: "if night-lights cause nearsightedness, the effect disappears with age."

LR bible had section for weaken X condition and weaken X cause-effect, and I encountered many of them.

This time, I was confused whether I was supposed to attack necessary condition or find an alternative explanation (one of five ways to attack cause-effect conclusion).

Then, even finding an alternative explanation was challenging since I was not sure whether I could just attack sufficient condition as well as necessary condition.

The spectrum of possible options broadened dramatically, and I was lost.

Q: Is there priority of the target to attack when conditional and causal statements are mixed in weaken question?

the reasons that I found my incorrect answers convincing:

answer A) it attacks causal relationship obviously. the only thing that concerned me was that this causal statement was included in sufficient condition of conclusion not necessary condition, and I don't know whether it makes difference if causal relationship is the primary target to attack.

answer E) It attacks necessary condition of conclusion. I was even more convinced by this answer than A. The conclusion states that the effect disappears with age, but answer E says even older children has the effect of being nearsighted after having slept with night-lights as infants. Still, I do not know why this answer is incorrect. The only flaw could relay on the subjects of 100 children with different variables affecting them possibly. (Is it really big flaw though?)

Answer D is the correct answer.
I do not know what exactly it does.

Is it attacking causal relationship between night-lights and nearshightedness thus the sufficient condition of conclusion?

Then, it is not different from the answer A.

Is it attacking the necessary condition of the effect disappearing with age by saying that the effect did not disappear, but was not found due to insufficient research?

I can be convinced of correct answer D but both answer A and E are too appealing to ignore.

I need help to find the reasons why they are incorrect.
 Adam Tyson
PowerScore Staff
  • PowerScore Staff
  • Posts: 5153
  • Joined: Apr 14, 2011
|
#14748
I hope I don't blow your mind with this reply, but I believe that the answer to whether you should go after the conditionality or the causality first is, in this case, neither. Instead, since the conclusion is based on studies, go after the validity of the studies themselves.

When an LSAT author bases a conclusion on a survey or study, my first thought is to question whether any conclusion of any kind can be reliably made based on the study or survey, or if it is too flawed to rely on. That's where answer D comes in - it tells us that the later studies were no good, that you can't rely on them for anything. The first study, showing a correlation between night lights and nearsightedness might still be good, but any conclusion about the effects disappearing with age are based solely on the validity of the later studies, which answer D tells us are trash. That weakens the conclusion pretty decisively.

Remember that when you weaken an argument you are not trying to disprove the conclusion - you just want to make it less likely, perhaps by showing that the premises don't support it. Could it be true? Sure, but this argument doesn't help it as much as the author would like to think.

So what about when you do see both causality and conditionality in an argument? I don't think there is any real formula for what comes first. Focus instead on the errors made by the author - did he make a mistaken reversal or mistaken negation? Other than pointing out one of those, the only way to weaken a conditional argument is to show that, at least once, the sufficient condition occurs and the necessary doesn't. If you have that situation, then you can weaken conditionally. Without that, if the argument makes a causal conclusion, focus on the classic causal problems - alternate cause, cause with out effect, etc.
 reop6780
  • Posts: 265
  • Joined: Jul 27, 2013
|
#14808
Hi, Adam.

When you say attacking the study, should the correct answer mention the study presented by the stimuli only?

Answer A and E also attack the study in the stimuli by additional study.

Are answer A and E wrong because they brought a further study and did not address the study done in the stimuli?

Or, do they fail to attack the conclusion in general?
If so, in what aspects?

Thank you

Get the most out of your LSAT Prep Plus subscription.

Analyze and track your performance with our Testing and Analytics Package.